LSAT and Law School Admissions Forum

Get expert LSAT preparation and law school admissions advice from PowerScore Test Preparation.

User avatar
 Dave Killoran
PowerScore Staff
  • PowerScore Staff
  • Posts: 5852
  • Joined: Mar 25, 2011
|
#42382
Complete Question Explanation
(The complete setup for this game can be found here: lsat/viewtopic.php?t=16021)

The correct answer choice is (B)

This question stem can be classified as a Justify, which appears infrequently in the Logic Games section. In this case, the answer choice, when added to the initial rules of the game, must force a situation in which no possible hypothetical can meet all of the conditions of the game. Since we know table 3 is the most restricted point in the game, it is a good bet that the answer choice will in some way affect the sponsors available to meet the table 3 requirements. This question stem can be classified as a Justify, which appears infrequently in the Logic Games section. In this case, the answer choice, when added to the initial rules of the game, must force a situation in which no possible hypothetical can meet all of the conditions of the game. Since we know table 3 is the most restricted point in the game, it is a good bet that the answer choice will in some way affect the sponsors available to meet the table 3 requirements. This occurs in answer choice (B), the correct answer choice, which places P and Q at tables 1 or 2, leaving only Z to fulfill the requirement that two sponsors sit at table 3.

This question also provides an excellent example of the technique of using hypotheticals from other questions to eliminate incorrect answer choices. For example, answer choice (A) states that “At most two sponsors are seated at table 1.” From question #6 we know that this scenario produces several workable hypotheticals. Since any workable hypothetical would conflict with the desired result of question #7, answer choice (A) must be wrong. Answer choice (D) can be eliminated by the same process. Answer choice (D) states, “Exactly three of the sponsors are seated at table 1.” The information in the stem from question #4 and also answer choice (D) in question #1 shows that this occurrence also allows several workable hypotheticals. Thus, answer choice (D) is incorrect. Answer choice (E) can be eliminated by examining the hypotheticals from question #3 as well as answer choice (D) in question #1. Remember, when a working hypothetical from another question meets the required conditions of a particular question, that hypothetical can be used to eliminate wrong answers or confirm the correct answer.
 jbeyer
  • Posts: 6
  • Joined: Jul 18, 2011
|
#881
Hi everyone,

I was doing game 1 questions number 7 and I did not know why C was not a possible answer. The answer key did not explain why C was not a valid option.
C: Any sponsor giving a speech is seated at table 2 or else table 3. Wouldn't that would force M (which is a speech and honors) to possible be at table 3. Wouldn't that violate the rule states that any person receiving honors must be seated at table 1 or 2?

Thanks so much,

Jonathan
User avatar
 Dave Killoran
PowerScore Staff
  • PowerScore Staff
  • Posts: 5852
  • Joined: Mar 25, 2011
|
#918
Hey Jonathan,

Thanks for the question. I'm not sure if you are misreading the answer and missed the "at table 2 or else 3" in answer choice (C), but there is no reason that M would have only table 3 as an option as you state; M could be seated at table 2 instead, where it would satisfy both answer choice (C) and the second rule. When that occurs, it is possible to make a workable seating assignment.

Does that make sense?

Get the most out of your LSAT Prep Plus subscription.

Analyze and track your performance with our Testing and Analytics Package.